LSAT and Law School Admissions Forum

Get expert LSAT preparation and law school admissions advice from PowerScore Test Preparation.

 stephaniemaui
  • Posts: 8
  • Joined: Jul 05, 2018
|
#47652
Hello,

I am hoping you can explain to me why B is incorrect and E is correct. I've read LSAC's explanation and chose B on both my inital testing and blind review.

The critic says that the participants who were favorably disposed to the substitution failed. I am thinking that B is incorrect now because the health association nor the critic state anything about if a person is more favorably disposed to the meatless regimen that they will be more likely to succeed that one is who not. The critic just merely states that even those participants who were favorably disposed failed to make the transition.

However, I still do not understand why E is correct. The critic doesn't seem to call into question whether people would or would not succeed in the regimen if the doctor told them to.

Thanks for your help in advance.
 lsat_novice
  • Posts: 29
  • Joined: May 29, 2018
|
#47754
I too selected B instead of E. I thought that B was correct because it relates to what the Critic is saying about being "favorably disposed." Any insight would be appreciated.

Also, I had a really hard time understanding what the question stem was asking for. What type of question is this?

Thank you!
 Jennifer Janowsky
PowerScore Staff
  • PowerScore Staff
  • Posts: 90
  • Joined: Aug 20, 2017
|
#47781
LSAT_Novice and Stephanie,

Thanks for your questions! Looks like both of you are having a similar issue.

To summarize, the Health Association says people volunteered to exclude meat from their diet and succeeded without problems, therefore people can change to meatless diets without suffering. A critic argues that these people were predisposed to give up meat beforehand, and that still many of them failed.

The stem asks the following: Using the same evidence from the stimulus, what answer choice does the critic's response most weaken? This is just a Cannot Be True question, in which you are seeing which answer choice doesn't follow.

(B) is designed to trick you, because it is actually the answer that most follows the evidence in the stimulus. The problem is, it is not an idea weakened by the critic's response. The critic's response would support that "more favorably disposed to the meatless regimen a person is the more likely that person is to succeed in the attempt," rather than weaken it. This means this answer is not a Cannot Be True--it is really the opposite.

Answer (E), however, is the following: Most people told to exclude meat would succeed in doing so. This follows from the evidence from the Health Association in the stimulus, while still being weakened by the critic's view.

I hope that clears up your questions, let me know if that helps!
User avatar
 ashpine17
  • Posts: 321
  • Joined: Apr 06, 2021
|
#90259
I don't understand the last response...wasn't the critic saying that many of those people who were favorably disposed to the meatless diet failed in their attempts? so how wouldn't that weaken what B is saying? I also don't understand the use of the word "many" here. I liked E but had doubts since it said "most" and I wasn't sure if a statement with "many" could weaken that
User avatar
 ashpine17
  • Posts: 321
  • Joined: Apr 06, 2021
|
#90262
Can I say B is wrong because I don't think that is a conclusion the health association would draw? They mention nothing about being predisposed to succeeding on meatless diet.
 Robert Carroll
PowerScore Staff
  • PowerScore Staff
  • Posts: 1783
  • Joined: Dec 06, 2013
|
#90434
ash,

But the critic is not claiming that trying the meatless diet made people more likely to fail. The critic is pointing out that some of the people who tried the meatless diet failed, so there was not universal success. The critic also thinks that the relative success (not universal, as I just pointed out) of those people had a good chance of being due to the fact that they were already inclined to adopt a meatless diet anyway. The critic is saying that the group is not representative - the people in the group were already more willing to give up meat than the average person.

It's perfectly compatible with the critic's statement that the more willing you are to give up meat, the more you will succeed. As I said, it's not like the "many" exceptions were more numerous than I would expect of a random person. So answer choice (B) is perfectly compatible with the critic's argument.

The "many" statement isn't being used at all to weaken the "most". It's the unrepresentative sample issue the critic points out that is weakening answer choice (E). So issues as to whether "many" can weaken "most" are not relevant here.

Robert Carroll
User avatar
 ashpine17
  • Posts: 321
  • Joined: Apr 06, 2021
|
#90454
Is answer choice B something the health association would agree with?
User avatar
 ashpine17
  • Posts: 321
  • Joined: Apr 06, 2021
|
#90455
Not B, I meant E!
 Rachael Wilkenfeld
PowerScore Staff
  • PowerScore Staff
  • Posts: 1358
  • Joined: Dec 15, 2011
|
#90729
Let me turn this back to you, Ash. Are we looking for something that the health association would definitely agree with? The question stem just says that the answer choices are possible conclusions to draw based on the health association's evidence. Not that they would be good, fully supported conclusions. Not that the health association would agree with the conclusions. Just that they would be potentially something one would try to draw based on the evidence given.

It's pizza night at my house, so I'm feeling like a pizza based example might help here.

Pretend we have this evidence: Every time I get pizza from D's Pizza Spot they mess up my order.

You could draw several conclusions based on that: I should stop getting pizza from D's. I should order something simpler from D's. Pizza night is a bad idea and should be forever canceled. Some of those I agree with. Some I don't. They are all possible conclusions to draw from my evidence.

Anyway, back to the LSAT question at hand. We don't need to know that the Health Association would be on board with the conclusions suggested in the answer choices. We just need to know that they conflict with the critic's statements on the topic.

Hope that helps!

Get the most out of your LSAT Prep Plus subscription.

Analyze and track your performance with our Testing and Analytics Package.